- PowerScore Staff
- Posts: 5972
- Joined: Mar 25, 2011
- Sat Jan 21, 2012 12:00 am
#43444
Complete Question Explanation
(The complete setup for this game can be found here: lsat/viewtopic.php?t=16406)
The correct answer choice is (C)
The question stem forms a FJH super-block. This block is obviously quite space-consuming, and the space restriction should be your starting point. If the FJH block is in group 1, then L must also be in group 1 from the third rule, which is impossible due to space restrictions (again, note how important it is to show the G/M dual option on the diagram). Thus, the FJH block must be in group 2, and the other three variables—K, L, and N—must be in group 1. Consequently, answer choice (C) is correct.
(The complete setup for this game can be found here: lsat/viewtopic.php?t=16406)
The correct answer choice is (C)
The question stem forms a FJH super-block. This block is obviously quite space-consuming, and the space restriction should be your starting point. If the FJH block is in group 1, then L must also be in group 1 from the third rule, which is impossible due to space restrictions (again, note how important it is to show the G/M dual option on the diagram). Thus, the FJH block must be in group 2, and the other three variables—K, L, and N—must be in group 1. Consequently, answer choice (C) is correct.
Dave Killoran
PowerScore Test Preparation
Follow me on X/Twitter at http://twitter.com/DaveKilloran
My LSAT Articles: http://blog.powerscore.com/lsat/author/dave-killoran
PowerScore Podcast: http://www.powerscore.com/lsat/podcast/
PowerScore Test Preparation
Follow me on X/Twitter at http://twitter.com/DaveKilloran
My LSAT Articles: http://blog.powerscore.com/lsat/author/dave-killoran
PowerScore Podcast: http://www.powerscore.com/lsat/podcast/